LSAT and Law School Admissions Forum

Get expert LSAT preparation and law school admissions advice from PowerScore Test Preparation.

User avatar
 Dave Killoran
PowerScore Staff
  • PowerScore Staff
  • Posts: 5853
  • Joined: Mar 25, 2011
|
#88685
Complete Question Explanation
(The complete setup for this game can be found here: lsat/viewtopic.php?f=229&p=88682)

The correct answer choice is (E)

If F, L, Q, and R are selected, then from the rules the following inferences can be made:

Second rule: Because Q and R are already selected, from the contrapositive of the second rule we can infer that no other biologist can be selected aside from F, who is already selected. This information eliminates answer choices (A) and (B).
Third rule: Because F is already selected, from this rule K cannot be selected, eliminating answer choice (C).
Fifth rule: If M were selected, then P and R must be selected. But, in this question, if M is selected, that would fill the panel, leaving no room for R. Thus, M cannot be selected, and answer choice (D) can be eliminated.
Thus, answer choice (E) is proven correct by process of elimination.
 rwitt09
  • Posts: 6
  • Joined: Oct 01, 2022
|
#102259
But R is already selected. To me it seems that both M and P must be selected, so I did not know which answer to pick. They both could go 5th to me because those are the only two left to select.
 Adam Tyson
PowerScore Staff
  • PowerScore Staff
  • Posts: 5153
  • Joined: Apr 14, 2011
|
#102282
If M is selected, rwitt09, then P must ALSO be selected, making our panel too big. FLQRMP is 6 people instead of the group of 5 we are supposed to choose. Since that cannot happen, M cannot be the last one selected. But we can select P without M, because although M requires P and R, P and R do not require M. That would be a Mistaken Reversal of that rule.

Get the most out of your LSAT Prep Plus subscription.

Analyze and track your performance with our Testing and Analytics Package.